Diễn Đàn MathScope

Diễn Đàn MathScope (http://forum.mathscope.org/index.php)
-   2014 (http://forum.mathscope.org/forumdisplay.php?f=177)
-   -   [VMO 2014] Bài 6 - Đại số (http://forum.mathscope.org/showthread.php?t=46339)

huynhcongbang 04-01-2014 11:49 AM

[VMO 2014] Bài 6 - Đại số
 
Bài 6. (7 điểm)

Tìm giá trị lớn nhất của biểu thức $$P=\frac{{{x}^{3}}{{y}^{4}}{{z}^{3}}}{({{x}^{4}}+ {{y}^{4}}){{(xy+{{z}^{2}})}^{3}}}+\frac{{{y}^{3}}{ {z}^{4}}{{x}^{3}}}{({{y}^{4}}+{{z}^{4}}){{(yz+{{x} ^{2}})}^{3}}}+\frac{{{z}^{3}}{{x}^{4}}{{y}^{3}}}{( {{z}^{4}}+{{x}^{4}}){{(zx+{{y}^{2}})}^{3}}}$$ với $x,y,z$ là các số thực dương.

huynhcongbang 04-01-2014 12:44 PM

2 Attachment(s)
Đây là lời giải của anh Cẩn cho bài BĐT này. Mọi người xem thử nhé! :)

[Only registered and activated users can see links. Click Here To Register...]

[Only registered and activated users can see links. Click Here To Register...]

Kelacloi 04-01-2014 01:02 PM

Giải như anh cẩn thì có thể thấy là bất đẳng thức năm nay hơi lỏng @.@, nhưng với điều kiện là nghĩ ra được lời giải như anh Cẩn

CSS-MU 04-01-2014 01:04 PM

Trích:

Nguyên văn bởi huynhcongbang (Post 199195)
Bài 6. (7 điểm)

Tìm giá trị lớn nhất của biểu thức $$P=\frac{{{x}^{3}}{{y}^{4}}{{z}^{3}}}{({{x}^{4}}+ {{y}^{4}}){{(xy+{{z}^{2}})}^{3}}}+\frac{{{y}^{3}}{ {z}^{4}}{{x}^{3}}}{({{y}^{4}}+{{z}^{4}}){{(yz+{{x} ^{2}})}^{3}}}+\frac{{{z}^{3}}{{x}^{4}}{{y}^{3}}}{( {{z}^{4}}+{{x}^{4}}){{(zx+{{y}^{2}})}^{3}}}$$ với $x,y,z$ là các số thực dương.

Thử phát :feelgood:
Đặt $\frac{x}{y}=a^2,\frac{y}{z}=b^2,\frac{z}{x}=c^2,$ $a,b,c>0$ thì $abc=1$ và ta sẽ chứng minh
$$\frac{1}{(a^8+1)(b^2+c^2)^3}+\frac{1}{(b^8+1)(c^ 2+a^2)^3}+\frac{1}{(c^8+1)(a^2+b^2)^3}\le\frac{3}{ 16}.$$
Sau khi dùng bất đẳng thức AM-GM thì việc còn lại của ta là chứng minh
$$\frac{1}{b^3c^3(a^8+1)}+\frac{1}{c^3a^3(b^8+1)}+ \frac{1}{a^3b^3(c^8+1)}\le\frac{3}{2},$$
hay là
$$\frac{a^3}{a^8+1}+\frac{b^3}{b^8+1}+\frac{c^3}{c ^8+1}\le 1.$$
Bây giờ ta sẽ chứng minh
$$\frac{a^3}{a^8+1}\le\frac{3(a^2+1)}{4(a^4+a^2+1) }.$$
Thật vậy, bằng biến đổi tương đương ta thấy nó tương đương với
$$(a-1)^2\left\{\left[(3a^5+5a^2+4)(a+1)+6a^4\right](a^2+a+1)-(a+1)^2\right\}\ge0,$$
hiển nhiên đúng. Thiết lập tương tự cho $b,c$ ta phải chứng minh
$$\sum\frac{3(a^2+1)}{4(a^4+a^2+1)}\le\frac{3}{2} \iff \sum\frac{1}{\frac{1}{a^4}+\frac{1}{a^2}+1}\ge 1,$$
là một kết quả quen thuộc. Vậy giá trị cần tìm là $1$ với đẳng thức khi và chỉ khi $a=b=c=1.$

K.I.A 04-01-2014 01:16 PM

Dễ chứng minh :$a^4+b^4 \ge \dfrac{2}{3}ab\left(a^2+b^2+ab\right)$ và $(a+b)^3 \ge 4ab(a+b)$
Áp dụng, ta được:
$\dfrac{x^3y^4z^3}{(x^4+y^4)(xy+z^2)^3} \le \dfrac{3}{8}.\dfrac{x^3y^4z^3}{xy(x^2+y^2+xy)z^2xy (z^2+xy)}$
$=\dfrac{3}{8}.\dfrac{xy^2z}{(x^2+xy+y^2)(z^2+xy)} $
$=\dfrac{3}{8}.\dfrac{xy^2z}{(x^2y^2+y^2z^2+z^2x^2 )+xy(x^2+y^2+z^2)}$
$\le \dfrac{3xy^2z}{32}.\left[\dfrac{1}{x^2y^2+y^2z^2+z^2x^2}+\dfrac{1}{xy(x^2+y ^2+z^2)}\right]$
$= \dfrac{3}{32}.\left(\dfrac{xyz.y}{x^2y^2+y^2z^2+z^ 2x^2}+\dfrac{yz}{x^2+y^2+z^2}\right)$
Đánh giá tương tự ta thu được:
$$T \le \dfrac{3}{32}.\left[\dfrac{xyz(x+y+z)}{x^2y^2+y^2z^2+z^2x^2}+\dfrac{xy +yz+zx}{x^2+y^2+z^2}\right] \le \dfrac{3}{16}$$
Cầu trời cho đúng +_+

vinhhop.qt 04-01-2014 01:25 PM

Trích:

Nguyên văn bởi K.I.A (Post 199226)
Dễ chứng minh :$a^4+b^4 \ge \dfrac{2}{3}ab\left(a^2+b^2+ab\right)$ và $(a+b)^3 \ge 4ab(a+b)$
Áp dụng, ta được:
$\dfrac{x^3y^4z^3}{(x^4+y^4)(xy+z^2)^3} \le \dfrac{3}{8}.\dfrac{x^3y^4z^3}{xy(x^2+y^2+xy)z^2xy (z^2+xy)}$
$=\dfrac{3}{8}.\dfrac{xy^2z}{(x^2+xy+y^2)(z^2+xy)} $
$=\dfrac{3}{8}.\dfrac{xy^2z}{(x^2y^2+y^2z^2+z^2x^2 )+xy(x^2+y^2+z^2)}$
$\le \dfrac{3xy^2z}{32}.\left[\dfrac{1}{x^2y^2+y^2z^2+z^2x^2}+\dfrac{1}{xy(x^2+y ^2+z^2)}\right]$
$= \dfrac{3}{32}.\left(\dfrac{xyz.y}{x^2y^2+y^2z^2+z^ 2x^2}+\dfrac{yz}{x^2+y^2+z^2}\right)$
Đánh giá tương tự ta thu được:
$$T \le \dfrac{3}{32}.\left[\dfrac{xyz(x+y+z)}{x^2y^2+y^2z^2+z^2x^2}+\dfrac{xy +yz+zx}{x^2+y^2+z^2}\right] \le \dfrac{3}{16}$$
Cầu trời cho đúng +_+

Đáp án là đây chăng?

hakudoshi 04-01-2014 01:38 PM

Trích:

Nguyên văn bởi K.I.A (Post 199226)
Dễ chứng minh :$a^4+b^4 \ge \dfrac{2}{3}ab\left(a^2+b^2+ab\right)$ và $(a+b)^3 \ge 4ab(a+b)$

Tôi đã khóc khi nhìn thấy cách giải này. :!
Chỗ $(a+b)^3 \ge 4ab(a+b)$ thì mình đánh giá được.

Còn $a^4+b^4 \ge \dfrac{2}{3}ab(a^2+b^2+ab) \Leftrightarrow (a-b)^2(3a^2+3b^2+4ab) \geq 0$ sao đánh giá hay vậy bạn :O

K.I.A 04-01-2014 01:45 PM

Trích:

Nguyên văn bởi hakudoshi (Post 199234)
Tôi đã khóc khi nhìn thấy cách giải này. :!
Chỗ $(a+b)^3 \ge 4ab(a+b)$ thì mình đánh giá được.
Còn $a^4+b^4 \ge \dfrac{2}{3}ab\left(a^2+b^2+ab\right)$ sao đánh giá hay vậy bạn :O

$\dfrac{3a^4+b^4}{2} \ge 2a^3b$
$\dfrac{a^4+3b^4}{2} \ge 2ab^3$
$a^4+b^4 \ge 2a^2b^2$
Cộng theo vế:
$3(a^4+b^4) \ge 2ab(a^2+b^2+ab)$

-----------------------------
$3a^2+3b^2+4ab=a^2+b^2+2(a+b)^2 \ge 0$

thaygiaocht 04-01-2014 02:02 PM

Trích:

Nguyên văn bởi huynhcongbang (Post 199195)
Bài 6. (7 điểm)

Tìm giá trị lớn nhất của biểu thức $$P=\frac{{{x}^{3}}{{y}^{4}}{{z}^{3}}}{({{x}^{4}}+ {{y}^{4}}){{(xy+{{z}^{2}})}^{3}}}+\frac{{{y}^{3}}{ {z}^{4}}{{x}^{3}}}{({{y}^{4}}+{{z}^{4}}){{(yz+{{x} ^{2}})}^{3}}}+\frac{{{z}^{3}}{{x}^{4}}{{y}^{3}}}{( {{z}^{4}}+{{x}^{4}}){{(zx+{{y}^{2}})}^{3}}}$$ với $x,y,z$ là các số thực dương.

Định hướng giải:
Quan sát phân thức thứ nhất thấy ở tử số $z $ có bậc 3, ở mẫu $z $ có bậc 6. Như vậy phản xạ đầu tiên là khử hết $z $ bằng AM-GM.
Cụ thể
$z^2+xy \ge 2z \sqrt{xy} $.
Như vậy, phân thức đầu tiên chỉ còn $x,y $ không có $z $ nữa.
Sau khi đặt $\sqrt{x}=a;... $ cho gọn ta thấy bài toán sẽ được giải quyết nếu có
$\dfrac{a^3b^5}{a^8+b^8}+\dfrac{b^3c^5}{b^8+c^8}+ \dfrac{c^3a^5}{c^8+a^8} \le \dfrac{3}{2}. $

Đến đây, ta lại đặt ẩn phụ để đưa về 1 biến $x=\dfrac{a}{b};... $ ta cần chứng minh
$f(x)+f(y)+f(z) \le \dfrac{3}{2} $ với $x,y,z>0 $ thỏa mãn $xyz=1 $ trong đó $f(x)=\dfrac{x^3}{x^8+1} $.

Đến đây có 2 hướng làm:
Hướng 1: Sử dụng đánh giá dạng
$\dfrac{x^3}{x^8+1} \le \dfrac{3}{4} \dfrac{x^k+1}{x^{2k}+x^k+1} $.
Sau đó dùng bất đẳng thức Vasc.
Cách tìm $k $:
$\dfrac{-2}{4}=\dfrac{3x^2(x^8+1)-8x^7.x^3}{(x^8+1)^2}=\dfrac{3}{4}.\dfrac{3k-3k.2}{9} $
Giải ra $k=2 $.
Hướng 2: Sử dụng đánh giá dạng
$\dfrac{x^3}{x^8+1} \le \dfrac{1}{2} + k \ln x $.
Sau đó nhân lại.
Cách tìm $k $:
$\dfrac{-2}{4}=\dfrac{3x^2(x^8+1)-8x^7.x^3}{(x^8+1)^2}=\dfrac{k}{x}=k $
Giải ra $k=\dfrac{-1}{2} $.

Ở hướng 1 thì $k=2 $ còn ở hướng 2 thì $k=\dfrac{-1}{2}. $
Hướng 1 được ngay, còn hướng 2 phải xét thêm trường hợp số lớn nhất lớn hơn $\dfrac{5}{2} $, chú ý không cần điều kiện gì thì $f(x) $ đã bị chặn trên. Trường hợp số lớn nhất lớn hơn $\dfrac{5}{2} $ cần đánh giá từng phân số rồi cộng lại.

Phụ lục:
1. (BĐT của Vasc) [Only registered and activated users can see links. Click Here To Register...]
2. (Bài liên quan) [Only registered and activated users can see links. Click Here To Register...]

Cụ thể:
- Hướng 1: Tùng đã làm ở trên.
- Hướng 2: Giả sử $x \le y \le z $.
+) Nếu $z \le \dfrac{5}{2} $ thì $0<x,y,z \le \dfrac{5}{2} $. Khi đó, ta sẽ chứng minh
$f(x) \le \dfrac{1-\ln x}{2} $
Thật vậy, xét $h(x)=f(x)-\dfrac{1-\ln x}{2} $ với $0<x \le \dfrac{5}{2} $.
Vẽ bảng biến thiên ta có $h(x) \le \max \{f(1); f \left( \dfrac{5}{2} \right) \}=0 $.
Do đó
$f(x) \le \dfrac{1-\ln x}{2} $
.
Suy ra
$f(x)+f(y)+f(z) \le \dfrac{3}{2}. $
+) Nếu $z > \dfrac{5}{2} $ thì $x \le \sqrt{xy}=\dfrac{1}{\sqrt{z}}<\sqrt{\dfrac{2}{5}} $.
Khi đó $f(x) < \dfrac{1}{4} $ do $x < \sqrt{\dfrac{2}{5}} $.
Lại có $f(z) < \dfrac{1}{50} $ do $z>\dfrac{5}{2} $.
và $f(y) < \dfrac{3}{5} $ (theo AM-GM).
Do vậy
$f(x)+f(y)+f(z) < \dfrac{3}{2}. $
Bài toán được giải quyết.

Lil.Tee 04-01-2014 02:17 PM

Trích:

Nguyên văn bởi Conanvn (Post 199223)
$$(x^4+y^4)(xy+z^2)^3 \ge xy(x^2+y^2).(4xyz^2)(xy+z^2)$$Nên $$T \le \dfrac{1}{4}\sum{\dfrac{xy^2z}{(x^2+y^2)(xy+z^2)}} $$$$=\dfrac{1}{4}\sum{\dfrac{xy^2z}{(x^2+y^2)xy+z^ 2(x^2+y^2)}}$$$$ \le \dfrac{1}{4} \sum{\dfrac{yz}{x^2+y^2+2z^2}}$$$$ \le \dfrac{1}{16} \sum{\left( \dfrac{y^2}{y^2+z^2}+\dfrac{z^2}{x^2+z^2} \right)}= \dfrac{3}{16}$$

Tổng cuối không bằng 3.

Traum 04-01-2014 06:27 PM

Ta có:
$(x^4 + y^4)\ge \frac{1}{8}(x+y)^4 $.
$(x+y)(xy+z^2) \ge (x\sqrt{y} + \sqrt{y}z)^2 = y(x+z)^2 $
$(x+y)(xy+z^2)\ge (\sqrt{x}y+\sqrt{x}z)^2 = x(y+z)^2 $
$xy+z^2\ge 2\sqrt{xy}{z} $
và $(x+y)(y+z)(z+x)\ge 8xyz $.

Vậy ta có: $(x^4 + y^4)(xy+z^2)^3\ge \frac{1}{4}(x+y)^2(y+z)^2(z+x)^2x^{3/2}y^{3/2}z $. Do đó:

$T\le 4\sum\frac{x^{3/2}y^{5/2}z^2}{(x+y)^2(y+z)^2(z+x)^2}\le \frac{1}{2}\sum\frac{x^{1/2}y^{3/2}z}{(x+y)(y+z)(z+x)} $.

Ta chứng minh $\sum x^{1/2}y^{3/2}z\le \frac{3}{8}(x+y)(y+z)(z+x) = \frac{3}{8}\left(2xyz+xy^2+x^2y+yz^2+y^2z+zx^2+z^2 x\right) $ (*).

Ta có: $x^2z + z^2y\ge xy^{1/2}z^{3/2} $, $z^2y + y^2x\ge 2x^{1/2}y^{3/2}z $ và $y^2x + x^2z\ge 2 x^{3/2}yz^{1/2} $. Hay $\sum xy^2\ge \sum x^{1/2}y^{3/2}z $.

Lại có: $y^2z + xyz\ge 2x^{1/2}y^{3/2}z $ nên ta có $\left(\sum y^2z\right) + 3xyz\ge 2\sum x^{1/2}y^{3/2}z $.

Vậy ta sẽ có:

$3xyz + \sum (xy^2+x^2y) \ge 3\sum x^{1/2}y^{3/2}z\ge \frac{8}{3}\left(\sum x^{1/2}y^{3/2}z\right) + xyz $ hay BĐT (*) được chứng minh.

Vậy ta luôn có $T\le \frac{1}{2}\frac{3}{8} = \frac{3}{16} $

truongvoki_bn 04-01-2014 08:44 PM

Trích:

Nguyên văn bởi huynhcongbang (Post 199195)
Bài 6. (7 điểm)

Tìm giá trị lớn nhất của biểu thức $$P=\frac{{{x}^{3}}{{y}^{4}}{{z}^{3}}}{({{x}^{4}}+ {{y}^{4}}){{(xy+{{z}^{2}})}^{3}}}+\frac{{{y}^{3}}{ {z}^{4}}{{x}^{3}}}{({{y}^{4}}+{{z}^{4}}){{(yz+{{x} ^{2}})}^{3}}}+\frac{{{z}^{3}}{{x}^{4}}{{y}^{3}}}{( {{z}^{4}}+{{x}^{4}}){{(zx+{{y}^{2}})}^{3}}}$$ với $x,y,z$ là các số thực dương.

Một hướng tiếp cận:
Đặt $\frac{x}{y}=a;\frac{y}{z}=b;\frac{z}{x}=c $
Ta được: $abc=1 $
Ta cần tìm GTLN của P= $\sum{\frac{1}{(b+c)^3(a^4+1)} $
Áp dụng Cauchy-Schwarz ta được $ P^2 \le (\sum{\frac{a^3}{(a^4+1)^2}).(\sum{\frac{1}{a^3(b+ c)^6}})\le \frac{3}{64}\sum{\frac{a^3}{(a^4+1)^2}} \le \frac{3}{128} (\sum{\frac{a^4}{(a^4+1)^2}+\sum{\frac{a^2}{(a^4+1 )^2}}) \le \frac{3}{128}(\frac{3}{4}+\sum{\frac{a^2}{(a^4+1)^ 2}}) $

Do vậy ta sẽ chỉ cần tìm GTLN của biểu thức $Q=\sum{\frac{a^2}{(a^4+1)^2}} $ với điều kiện $abc=1 $
Thay ngược trở x;y;z trở lại ta cần tìm GTLN của $Q=\sum{\frac{x^2y^6}{(x^4+y^4)^2}} $ với mọi số dương $x;y;z $
Áp dụng AM-GM ta được $Q \le \sum{\frac{y^4}{2\sqrt{2(x^8+y^8)}}} $
Ta chứng minh được bài toán sau bằng dồn biến:


Do đó $ Q\le \frac{3}{4} $

Từ đó ta được GTLN của $P=\frac{3}{16} $

luugiangnam 04-01-2014 09:23 PM

Có 1 hướng giải cũng khá hay( mới nghĩ ra, chưa biết đúng không ) .:gach: .

Ta có $a^4+b^4\geq \frac{1}{2}(a^2+b^2)(a^2+b^2)\geq ab(a^2+b^2)$.

Từ đó $T\leq \sum \frac{x^2y^3z^3}{(x^2+y^2)(xy+z^2)^3}=\sum \frac{1}{(1+(\frac{y}{x})^2)(\frac{x}{z}+\frac{z}{ y})^3}$

Đặt $a=\frac{y}{x},b=\frac{x}{z}, c=\frac{z}{y}$ => $abc=1$.

Khi đó $T\leq \sum \frac{1}{(1+a^2)(b+c)^3}=\sum \frac{1}{(1+a^2)(b+c)^2(b+c)}\leq \sum \frac{1}{16abc.\sqrt{bc}}=\frac{1}{16}.\sum \sqrt{a}$ .

Vì $abc=1$ nên T là đa thức thuần nhất => Chuẩn hóa $a+b+c=3$.

Vậy $T\leq \frac{1}{16}.\sqrt{3(a+b+c)}=\frac{3}{16}$ khi $a=b=c=1$ hay $x=y=z$.

Mong mọi người góp ý.

daicahuyvn 04-01-2014 09:41 PM

$\frac{x^3y^4z^3}{(x^4+y^4)(xy+z^2)^3}\le \frac{x^3y^4z^3}{8xy(x^2+y^2)z^3xy\sqrt{xy}}=\frac {y\sqrt{xy}}{8(x^2+y^2)}$
$a=\sqrt{\frac{x}{y}}$.abc=1
$8T\le \sum \frac{a}{a^4+1}\le \sum \frac{1}{\sqrt{2(a^4+1)}}\le\frac{3}{2}$

Lil.Tee 04-01-2014 09:55 PM

Trích:

Nguyên văn bởi luugiangnam (Post 199292)
Vì $abc=1$ nên T là đa thức thuần nhất => Chuẩn hóa $a+b+c=3$.

Đoạn này mình nghĩ không được, vì nếu bạn đã sử dụng $abc=1$ mà lại chuẩn hóa $a+b+c=3$ thì có khác nào $a=b=c=1$ luôn?


Múi giờ GMT. Hiện tại là 03:48 AM.

Powered by: vBulletin Copyright ©2000-2024, Jelsoft Enterprises Ltd.

[page compression: 25.31 k/26.79 k (5.54%)]